Bound for $e^{-alpha x}$












1












$begingroup$


For a part of my proof I need to establish that $e^{-alpha x} lt h(x)$, where $alpha,x gt 0, $ and $x,alpha inmathbb{R}$. I thought for a while and couldn't find a function independent of $alpha$ that fulfills my criteria. Any ideas?










share|cite|improve this question











$endgroup$








  • 1




    $begingroup$
    Are you asking for an example of a function $h$ for which the inequality is true?
    $endgroup$
    – kimchi lover
    Jan 15 at 14:21






  • 2




    $begingroup$
    $h(x)=1$ fits the bill.
    $endgroup$
    – Adrian Keister
    Jan 15 at 14:31






  • 1




    $begingroup$
    If you don't want any specific $h$, there are a lot (and basically useless) functions that bound $e^{-ax}$ from above. For example given any strictly positive integer $n$, $h_n(x)=n$ is always strictly larger than $e^{-ax}$ and independent of $a$.
    $endgroup$
    – James
    Jan 15 at 14:36












  • $begingroup$
    I'm voting to close the question for lack of context. Without context, we cannot know what kinda of upper bound you want, and the question becomes too open-ended.
    $endgroup$
    – Simply Beautiful Art
    Jan 15 at 14:37






  • 1




    $begingroup$
    $$ alpha,x>0implies e^{-alpha x}<1.$$
    $endgroup$
    – Julián Aguirre
    Jan 15 at 15:15
















1












$begingroup$


For a part of my proof I need to establish that $e^{-alpha x} lt h(x)$, where $alpha,x gt 0, $ and $x,alpha inmathbb{R}$. I thought for a while and couldn't find a function independent of $alpha$ that fulfills my criteria. Any ideas?










share|cite|improve this question











$endgroup$








  • 1




    $begingroup$
    Are you asking for an example of a function $h$ for which the inequality is true?
    $endgroup$
    – kimchi lover
    Jan 15 at 14:21






  • 2




    $begingroup$
    $h(x)=1$ fits the bill.
    $endgroup$
    – Adrian Keister
    Jan 15 at 14:31






  • 1




    $begingroup$
    If you don't want any specific $h$, there are a lot (and basically useless) functions that bound $e^{-ax}$ from above. For example given any strictly positive integer $n$, $h_n(x)=n$ is always strictly larger than $e^{-ax}$ and independent of $a$.
    $endgroup$
    – James
    Jan 15 at 14:36












  • $begingroup$
    I'm voting to close the question for lack of context. Without context, we cannot know what kinda of upper bound you want, and the question becomes too open-ended.
    $endgroup$
    – Simply Beautiful Art
    Jan 15 at 14:37






  • 1




    $begingroup$
    $$ alpha,x>0implies e^{-alpha x}<1.$$
    $endgroup$
    – Julián Aguirre
    Jan 15 at 15:15














1












1








1


0



$begingroup$


For a part of my proof I need to establish that $e^{-alpha x} lt h(x)$, where $alpha,x gt 0, $ and $x,alpha inmathbb{R}$. I thought for a while and couldn't find a function independent of $alpha$ that fulfills my criteria. Any ideas?










share|cite|improve this question











$endgroup$




For a part of my proof I need to establish that $e^{-alpha x} lt h(x)$, where $alpha,x gt 0, $ and $x,alpha inmathbb{R}$. I thought for a while and couldn't find a function independent of $alpha$ that fulfills my criteria. Any ideas?







exponential-function functional-inequalities






share|cite|improve this question















share|cite|improve this question













share|cite|improve this question




share|cite|improve this question








edited Jan 15 at 14:32









Adrian Keister

5,23271933




5,23271933










asked Jan 15 at 14:14









Michael MaierMichael Maier

859




859








  • 1




    $begingroup$
    Are you asking for an example of a function $h$ for which the inequality is true?
    $endgroup$
    – kimchi lover
    Jan 15 at 14:21






  • 2




    $begingroup$
    $h(x)=1$ fits the bill.
    $endgroup$
    – Adrian Keister
    Jan 15 at 14:31






  • 1




    $begingroup$
    If you don't want any specific $h$, there are a lot (and basically useless) functions that bound $e^{-ax}$ from above. For example given any strictly positive integer $n$, $h_n(x)=n$ is always strictly larger than $e^{-ax}$ and independent of $a$.
    $endgroup$
    – James
    Jan 15 at 14:36












  • $begingroup$
    I'm voting to close the question for lack of context. Without context, we cannot know what kinda of upper bound you want, and the question becomes too open-ended.
    $endgroup$
    – Simply Beautiful Art
    Jan 15 at 14:37






  • 1




    $begingroup$
    $$ alpha,x>0implies e^{-alpha x}<1.$$
    $endgroup$
    – Julián Aguirre
    Jan 15 at 15:15














  • 1




    $begingroup$
    Are you asking for an example of a function $h$ for which the inequality is true?
    $endgroup$
    – kimchi lover
    Jan 15 at 14:21






  • 2




    $begingroup$
    $h(x)=1$ fits the bill.
    $endgroup$
    – Adrian Keister
    Jan 15 at 14:31






  • 1




    $begingroup$
    If you don't want any specific $h$, there are a lot (and basically useless) functions that bound $e^{-ax}$ from above. For example given any strictly positive integer $n$, $h_n(x)=n$ is always strictly larger than $e^{-ax}$ and independent of $a$.
    $endgroup$
    – James
    Jan 15 at 14:36












  • $begingroup$
    I'm voting to close the question for lack of context. Without context, we cannot know what kinda of upper bound you want, and the question becomes too open-ended.
    $endgroup$
    – Simply Beautiful Art
    Jan 15 at 14:37






  • 1




    $begingroup$
    $$ alpha,x>0implies e^{-alpha x}<1.$$
    $endgroup$
    – Julián Aguirre
    Jan 15 at 15:15








1




1




$begingroup$
Are you asking for an example of a function $h$ for which the inequality is true?
$endgroup$
– kimchi lover
Jan 15 at 14:21




$begingroup$
Are you asking for an example of a function $h$ for which the inequality is true?
$endgroup$
– kimchi lover
Jan 15 at 14:21




2




2




$begingroup$
$h(x)=1$ fits the bill.
$endgroup$
– Adrian Keister
Jan 15 at 14:31




$begingroup$
$h(x)=1$ fits the bill.
$endgroup$
– Adrian Keister
Jan 15 at 14:31




1




1




$begingroup$
If you don't want any specific $h$, there are a lot (and basically useless) functions that bound $e^{-ax}$ from above. For example given any strictly positive integer $n$, $h_n(x)=n$ is always strictly larger than $e^{-ax}$ and independent of $a$.
$endgroup$
– James
Jan 15 at 14:36






$begingroup$
If you don't want any specific $h$, there are a lot (and basically useless) functions that bound $e^{-ax}$ from above. For example given any strictly positive integer $n$, $h_n(x)=n$ is always strictly larger than $e^{-ax}$ and independent of $a$.
$endgroup$
– James
Jan 15 at 14:36














$begingroup$
I'm voting to close the question for lack of context. Without context, we cannot know what kinda of upper bound you want, and the question becomes too open-ended.
$endgroup$
– Simply Beautiful Art
Jan 15 at 14:37




$begingroup$
I'm voting to close the question for lack of context. Without context, we cannot know what kinda of upper bound you want, and the question becomes too open-ended.
$endgroup$
– Simply Beautiful Art
Jan 15 at 14:37




1




1




$begingroup$
$$ alpha,x>0implies e^{-alpha x}<1.$$
$endgroup$
– Julián Aguirre
Jan 15 at 15:15




$begingroup$
$$ alpha,x>0implies e^{-alpha x}<1.$$
$endgroup$
– Julián Aguirre
Jan 15 at 15:15










1 Answer
1






active

oldest

votes


















-1












$begingroup$

Also note that:
$$e^{-alpha x}=sum_{n=0}^inftyfrac{(-1)^nalpha^nx^n}{n!}$$
A polynomial of this expansion could be used, although take care to see that it is consistently greater as this is a series with alternating sign






share|cite|improve this answer









$endgroup$













  • $begingroup$
    But a polynomial (of positive degree) gives a worse bound than the obvious $h(x)=1$.
    $endgroup$
    – David C. Ullrich
    Jan 15 at 17:41










  • $begingroup$
    The higher order used the closer this approximation is surely, especially at smaller values of $x$, since $h(x)=1$ is most accurate for $xtoinfty$
    $endgroup$
    – Henry Lee
    Jan 15 at 18:53











Your Answer





StackExchange.ifUsing("editor", function () {
return StackExchange.using("mathjaxEditing", function () {
StackExchange.MarkdownEditor.creationCallbacks.add(function (editor, postfix) {
StackExchange.mathjaxEditing.prepareWmdForMathJax(editor, postfix, [["$", "$"], ["\\(","\\)"]]);
});
});
}, "mathjax-editing");

StackExchange.ready(function() {
var channelOptions = {
tags: "".split(" "),
id: "69"
};
initTagRenderer("".split(" "), "".split(" "), channelOptions);

StackExchange.using("externalEditor", function() {
// Have to fire editor after snippets, if snippets enabled
if (StackExchange.settings.snippets.snippetsEnabled) {
StackExchange.using("snippets", function() {
createEditor();
});
}
else {
createEditor();
}
});

function createEditor() {
StackExchange.prepareEditor({
heartbeatType: 'answer',
autoActivateHeartbeat: false,
convertImagesToLinks: true,
noModals: true,
showLowRepImageUploadWarning: true,
reputationToPostImages: 10,
bindNavPrevention: true,
postfix: "",
imageUploader: {
brandingHtml: "Powered by u003ca class="icon-imgur-white" href="https://imgur.com/"u003eu003c/au003e",
contentPolicyHtml: "User contributions licensed under u003ca href="https://creativecommons.org/licenses/by-sa/3.0/"u003ecc by-sa 3.0 with attribution requiredu003c/au003e u003ca href="https://stackoverflow.com/legal/content-policy"u003e(content policy)u003c/au003e",
allowUrls: true
},
noCode: true, onDemand: true,
discardSelector: ".discard-answer"
,immediatelyShowMarkdownHelp:true
});


}
});














draft saved

draft discarded


















StackExchange.ready(
function () {
StackExchange.openid.initPostLogin('.new-post-login', 'https%3a%2f%2fmath.stackexchange.com%2fquestions%2f3074479%2fbound-for-e-alpha-x%23new-answer', 'question_page');
}
);

Post as a guest















Required, but never shown

























1 Answer
1






active

oldest

votes








1 Answer
1






active

oldest

votes









active

oldest

votes






active

oldest

votes









-1












$begingroup$

Also note that:
$$e^{-alpha x}=sum_{n=0}^inftyfrac{(-1)^nalpha^nx^n}{n!}$$
A polynomial of this expansion could be used, although take care to see that it is consistently greater as this is a series with alternating sign






share|cite|improve this answer









$endgroup$













  • $begingroup$
    But a polynomial (of positive degree) gives a worse bound than the obvious $h(x)=1$.
    $endgroup$
    – David C. Ullrich
    Jan 15 at 17:41










  • $begingroup$
    The higher order used the closer this approximation is surely, especially at smaller values of $x$, since $h(x)=1$ is most accurate for $xtoinfty$
    $endgroup$
    – Henry Lee
    Jan 15 at 18:53
















-1












$begingroup$

Also note that:
$$e^{-alpha x}=sum_{n=0}^inftyfrac{(-1)^nalpha^nx^n}{n!}$$
A polynomial of this expansion could be used, although take care to see that it is consistently greater as this is a series with alternating sign






share|cite|improve this answer









$endgroup$













  • $begingroup$
    But a polynomial (of positive degree) gives a worse bound than the obvious $h(x)=1$.
    $endgroup$
    – David C. Ullrich
    Jan 15 at 17:41










  • $begingroup$
    The higher order used the closer this approximation is surely, especially at smaller values of $x$, since $h(x)=1$ is most accurate for $xtoinfty$
    $endgroup$
    – Henry Lee
    Jan 15 at 18:53














-1












-1








-1





$begingroup$

Also note that:
$$e^{-alpha x}=sum_{n=0}^inftyfrac{(-1)^nalpha^nx^n}{n!}$$
A polynomial of this expansion could be used, although take care to see that it is consistently greater as this is a series with alternating sign






share|cite|improve this answer









$endgroup$



Also note that:
$$e^{-alpha x}=sum_{n=0}^inftyfrac{(-1)^nalpha^nx^n}{n!}$$
A polynomial of this expansion could be used, although take care to see that it is consistently greater as this is a series with alternating sign







share|cite|improve this answer












share|cite|improve this answer



share|cite|improve this answer










answered Jan 15 at 16:40









Henry LeeHenry Lee

2,042219




2,042219












  • $begingroup$
    But a polynomial (of positive degree) gives a worse bound than the obvious $h(x)=1$.
    $endgroup$
    – David C. Ullrich
    Jan 15 at 17:41










  • $begingroup$
    The higher order used the closer this approximation is surely, especially at smaller values of $x$, since $h(x)=1$ is most accurate for $xtoinfty$
    $endgroup$
    – Henry Lee
    Jan 15 at 18:53


















  • $begingroup$
    But a polynomial (of positive degree) gives a worse bound than the obvious $h(x)=1$.
    $endgroup$
    – David C. Ullrich
    Jan 15 at 17:41










  • $begingroup$
    The higher order used the closer this approximation is surely, especially at smaller values of $x$, since $h(x)=1$ is most accurate for $xtoinfty$
    $endgroup$
    – Henry Lee
    Jan 15 at 18:53
















$begingroup$
But a polynomial (of positive degree) gives a worse bound than the obvious $h(x)=1$.
$endgroup$
– David C. Ullrich
Jan 15 at 17:41




$begingroup$
But a polynomial (of positive degree) gives a worse bound than the obvious $h(x)=1$.
$endgroup$
– David C. Ullrich
Jan 15 at 17:41












$begingroup$
The higher order used the closer this approximation is surely, especially at smaller values of $x$, since $h(x)=1$ is most accurate for $xtoinfty$
$endgroup$
– Henry Lee
Jan 15 at 18:53




$begingroup$
The higher order used the closer this approximation is surely, especially at smaller values of $x$, since $h(x)=1$ is most accurate for $xtoinfty$
$endgroup$
– Henry Lee
Jan 15 at 18:53


















draft saved

draft discarded




















































Thanks for contributing an answer to Mathematics Stack Exchange!


  • Please be sure to answer the question. Provide details and share your research!

But avoid



  • Asking for help, clarification, or responding to other answers.

  • Making statements based on opinion; back them up with references or personal experience.


Use MathJax to format equations. MathJax reference.


To learn more, see our tips on writing great answers.




draft saved


draft discarded














StackExchange.ready(
function () {
StackExchange.openid.initPostLogin('.new-post-login', 'https%3a%2f%2fmath.stackexchange.com%2fquestions%2f3074479%2fbound-for-e-alpha-x%23new-answer', 'question_page');
}
);

Post as a guest















Required, but never shown





















































Required, but never shown














Required, but never shown












Required, but never shown







Required, but never shown

































Required, but never shown














Required, but never shown












Required, but never shown







Required, but never shown







Popular posts from this blog

Can a sorcerer learn a 5th-level spell early by creating spell slots using the Font of Magic feature?

Does disintegrating a polymorphed enemy still kill it after the 2018 errata?

A Topological Invariant for $pi_3(U(n))$